¿Sabemos por qué el campo producido por la carga se extiende hasta el infinito?

¿Tenemos algún modelo para mostrar por qué el campo de carga o el campo magnético se extiende hasta el infinito?

Editar: acepto que de acuerdo con la ley de culombios 1 / r 2 no puede ser 0 pero sabemos por qué sucede esto. Creo que estoy buscando un modelo físico.

(corrígeme si estoy equivocado)

Vaya, buena pregunta. Me pregunto, que pasa con el campo cuando la carga cruza el valor del tablón, y/o la energía que podría transferir, también sería menor que la energía del tablón.. ¿Será posible registrar de alguna manera? ¿O eso significaría que el campo llegó a su límite?
+noncom Sería interesante saber (si podemos) por qué la magnitud se reduce con la distancia
Tendría que encontrar una forma de que las líneas de campo terminen en el espacio medio.
@Jon Custer Lea mi respuesta. Cualquier crítica es bienvenida.
@noncom ¿Ha obtenido nuevos conocimientos sobre este tema?
@HolgerFiedler no, no he encontrado ninguna mención de algo así. Pero creo que mi pregunta era incorrecta en sí misma, ya que la energía de Plank es en realidad un gran valor ( en.wikipedia.org/wiki/Planck_units#Planck_energy ). Así que probablemente debería haber usado otro término, en lugar de "menos de energía de tabla", debería haber preguntado "por debajo del umbral mínimo medible", y luego tendríamos la pregunta: ¿se puede medir el campo más allá de eso? y si 2 o más de esos campos se unen para subir el nivel por encima de ese mínimo, ¿seremos capaces de detectar eso o no? pero no lo se :)

Respuestas (3)

El campo eléctrico se extiende hasta el infinito en el sentido de que nunca se encontró un límite después del cual el campo desaparecería. Es una suposición natural que simplifica las cosas. La ley de Coulomb es consistente con esta suposición, pero no existe un modelo que explique la ley de Coulomb a partir de algo más simple.

francamente me hace sentir mal.

En realidad, tenemos modelos que explican esto. Las partículas que median el campo electromagnético no tienen masa, por lo que el rango de fuerza predicho por el modelo es infinito. Por el contrario, para los mediadores masivos (ver, por ejemplo, la fuerza de Yukawa), el rango es finito.

Tenga en cuenta que las configuraciones experimentales reales tienen una precisión finita, por lo que más allá de algún límite no tiene sentido realizar mediciones (sin mencionar el ruido, la incertidumbre cuántica, etc.).

No soy físico y llegué al problema de la infinidad del campo eléctrico trabajando en estructuras unidimensionales complejas en el espacio. En este trabajo reconocí que para construir campos dipolares se necesitan al menos dos cuantos diferentes . Tales estructuras podrían seguir la ley 1/r², PERO en relación con la estructura discreta, el campo tiene un rango finito. ¿Por qué esto no se observó hasta ahora? En primer lugar, los cuantos tienen que estar en un rango de energía mucho más pequeño que los fotones. En segundo lugar, la necesidad en la descripción de campos dipolares de fuerza cuantificados parece no existir hasta ahora. Pero su pregunta y los muy buenos comentarios a su pregunta me animan a responder su pregunta.

Las consecuencias del postulado sobre los campos dipolares de fuerza cuantizados son extrañas. Las líneas de campo de los campos dipolares tienen una realidad. El campo magnético, el campo eléctrico y la radiación electromagnética se construyen a partir de los mismos cuantos. Lo nombra después de leer mi elaboración sobre estructuras unidimensionales complejas en el espacio . El artículo original está en alemán: Komplex eindimensionale Strukturen des Raumes. Quantenstruktur der Photonen .

Las consecuencias son enormes y los docentes de este foro quizás no estén preparados para pensar en esto, quieren defender la corriente principal. Los científicos en este foro quizás estén ocupados con sus propios problemas y soluciones, pero ya que están aquí quizás presten atención y adjunten críticas.

No estoy en el tema de los problemas con la infinidad de campos dipolares de fuerza . Quizás la renormalización tenga que ver con este problema, pero esto es una especulación de un no especialista.
Además, aquí hay un enlace a una respuesta mía ocho veces rechazada physics.stackexchange.com/questions/168684/… .